Syvum Home Page

Home > GMAT Test Prep > Print Preview

GMAT Test Prep : Quantitative Data Sufficiency Test I

Formats Worksheet / Test Paper Quiz Review

Hide all answers   View all answers   Print   Try the Quiz

Is the data given in the two statements, labeled (1) and (2), sufficient for answering the question?
All numbers used are real numbers.


1. Is p = q?
(1) (p + q)2 / pq = 4
(2) p2q2 − 150p + 150q = 0
• Statement (1) ALONE is sufficient, but statement (2) ALONE is not sufficient.
• Statement (2) ALONE is sufficient, but statement (1) ALONE is not sufficient.
• BOTH statements TOGETHER are sufficient, but NEITHER statement ALONE is sufficient.
• EACH statement ALONE is sufficient.
• Statements (1) and (2) TOGETHER are NOT sufficient.
Answer: Statement (1) ALONE is sufficient, but statement (2) ALONE is not sufficient.
Statement (1) specifies that (p + q)2 = 4pq or p2 + q2 + 2pq = 4pq.
So, p2 + q2 − 2pq = 0.
Hence, (pq)2 = 0, which implies p = q.
Thus, statement (1) ALONE is sufficient.
Statement (2) specifies that (p + q)(pq) − 150(pq) = 0.
Hence, (pq)(p + q − 150) = 0, which implies p = q OR p + q = 150.
Thus, statement (2) ALONE is not sufficient.

2. Did the incumbent Governor get more than 50% of the votes cast by the public in the recall election (Challengers B and C were the only other candidates who fought the election)?
(1) Challenger B got 36% of the votes.
(2) Challenger C got 60000 out of a total of 100000 votes.
• Statement (1) ALONE is sufficient, but statement (2) ALONE is not sufficient.
• Statement (2) ALONE is sufficient, but statement (1) ALONE is not sufficient.
• BOTH statements TOGETHER are sufficient, but NEITHER statement ALONE is sufficient.
• EACH statement ALONE is sufficient.
• Statements (1) and (2) TOGETHER are NOT sufficient.
Answer: Statement (2) ALONE is sufficient, but statement (1) ALONE is not sufficient.
Statement (1) specifies the percentage of votes Challenger B got, but nothing about the votes Challenger C got. Thus, statement (1) ALONE is not sufficient.
Statement (2) indicates that Challenger C got 60% (i.e., 60000 / 100000 x 100%) of the votes cast, which implies that no other candidate could have secured more than 40% of the votes.
Thus, statement (2) ALONE is sufficient.

3. By what percentage did the price of a pound of tomatoes increase?
(1) Before the increase, the price of a pound of tomatoes was 50 cents.
(2) The price of tomatoes increased by 20 cents per pound.
• Statement (1) ALONE is sufficient, but statement (2) ALONE is not sufficient.
• Statement (2) ALONE is sufficient, but statement (1) ALONE is not sufficient.
• BOTH statements TOGETHER are sufficient, but NEITHER statement ALONE is sufficient.
• EACH statement ALONE is sufficient.
• Statements (1) and (2) TOGETHER are NOT sufficient.
Answer: BOTH statements TOGETHER are sufficient, but NEITHER statement ALONE is sufficient.
Statement (1) specifies the price of a pound of tomatoes before the increase, but nothing about the actual increase. Thus, statement (1) ALONE is not sufficient.
Statement (2) specifies the increase in the price, but nothing about the original price.
Thus, statement (2) ALONE is not sufficient.
When considered together, the percentage increase in the price is (20/50) × 100% = 40%.
Thus, BOTH statements TOGETHER are sufficient.


4. The laborers in a factory speak only Spanish or only French or both the languages. What fraction of the laborers speak only Spanish?
(1) One-third of the laborers in the factory speak both the languages.
(2) 300 laborers in the factory speak only French.
• Statement (1) ALONE is sufficient, but statement (2) ALONE is not sufficient.
• Statement (2) ALONE is sufficient, but statement (1) ALONE is not sufficient.
• BOTH statements TOGETHER are sufficient, but NEITHER statement ALONE is sufficient.
• EACH statement ALONE is sufficient.
• Statements (1) and (2) TOGETHER are NOT sufficient.
Answer: Statements (1) and (2) TOGETHER are NOT sufficient.
Let S = number of laborers who speak only Spanish, F = number of laborers who speak only French, B = number of laborers who speak both languages and T = total number of laborers. Then,
S + F + B = T.
∴ Fraction of laborers who speak only Spanish = (S/T) = 1 − (F/T) − (B/T).
Statement (1) specifies (B/T), but not (F/T).
Thus, statement (1) ALONE is not sufficient.
Statement (2) specifies only F, but neither B nor T.
Thus, statement (2) ALONE is not sufficient.
When both statements are considered, (B/T) and F are known. However, without knowing T, the required fraction cannot be calculated.
Thus, statements (1) and (2) TOGETHER are NOT sufficient.
This type of problem may be represented conveniently as two intersecting sets on a Venn diagram.

5. Is the largest of seven consecutive integers an odd number?
(1) The product of the integers is even.
(2) The sum of the integers is zero.
• Statement (1) ALONE is sufficient, but statement (2) ALONE is not sufficient.
• Statement (2) ALONE is sufficient, but statement (1) ALONE is not sufficient.
• BOTH statements TOGETHER are sufficient, but NEITHER statement ALONE is sufficient.
• EACH statement ALONE is sufficient.
• Statements (1) and (2) TOGETHER are NOT sufficient.
Answer: Statement (2) ALONE is sufficient, but statement (1) ALONE is not sufficient.
Statement (1) provides no useful information because the product of two or more consecutive integers is always even. Thus, statement (1) ALONE is not sufficient.
Statement (2) specifies that the sum of the integers is zero.
Let the seven consecutive integers be n, n + 1, ... , n + 6.
Then, 7n + 21 = 0 or n = −21/7 = −3.
The seven consecutive integers are −3, −2, −1, 0, 1, 2 and 3 (whose largest number is odd).
Thus, statement (2) ALONE is sufficient.
Alternative Approach :
The set of 7 consecutive integers can consist of 3 odd integers and 4 even integers, or 4 odd integers and 3 even integers.
If the set consists of 3 odd integers (whose sum is odd) and 4 even integers (whose sum is even), then the sum of the 7 integers cannot go to 0.
So, the set of 7 consecutive integers must consist of 4 odd integers (whose sum is even) and 3 even integers (whose sum is also even).
∴ The set has the form OEOEOEO where E represents an even integer and O represents an odd integer.
This implies that the largest number is odd. Thus, statement (2) ALONE is sufficient.

6. $50000 are distributed among P, Q and R. Who got the least amount?
(1) P received one-fourth the amount that Q and R received together.
(2) R received what P and Q received together.
• Statement (1) ALONE is sufficient, but statement (2) ALONE is not sufficient.
• Statement (2) ALONE is sufficient, but statement (1) ALONE is not sufficient.
• BOTH statements TOGETHER are sufficient, but NEITHER statement ALONE is sufficient.
• EACH statement ALONE is sufficient.
• Statements (1) and (2) TOGETHER are NOT sufficient.
Answer: BOTH statements TOGETHER are sufficient, but NEITHER statement ALONE is sufficient.
Let p, q and r be the amounts that P, Q and R received, respectively. Then, p + q + r = 50000.
Statement (1) gives p = (q + r) / 4. It is not possible to conclude who got the least amount from this equation, because there are many possibilities for p, q and r. For example, if p = 10000, then q and r can have values of 15000 and 25000, or5000 and 35000.Thus, statement (1) ALONE is not sufficient.
Statement (2) gives r = p + q. This implies that R got the highest amount. However, it is not possible to conclude who (P or Q) got the least amount.Thus, statement (2) ALONE is not sufficient.
However, if both statements are considered together, then the three equations may be solved for the three unknowns to obtain the following unique solution: p = 10000, q = 15000, and r = 25000. Therefore, P got the least amount.Thus, BOTH statements TOGETHER are sufficient.

7. Is the quadrilateral ABCD a parallelogram?
(1) Sides AB and BC are equal in length.
(2) Sides BC, CD and AD are equal in length.
• Statement (1) ALONE is sufficient, but statement (2) ALONE is not sufficient.
• Statement (2) ALONE is sufficient, but statement (1) ALONE is not sufficient.
• BOTH statements TOGETHER are sufficient, but NEITHER statement ALONE is sufficient.
• EACH statement ALONE is sufficient.
• Statements (1) and (2) TOGETHER are NOT sufficient.
Answer: BOTH statements TOGETHER are sufficient, but NEITHER statement ALONE is sufficient.
A parallelogram is a quadrilateral whose opposite sides are parallel. Note that the opposite sides and opposite angles of a parallelogram are equal.
Statement (1) specifies that 2 adjacent sides of the quadrilateral are equal.
However, nothing is specified about the other 2 sides, which need not be equal.
So, ABCD need not necessarily be a parallelogram.
Thus, statement (1) ALONE is not sufficient.
Statement (2) implies that 3 sides of the quadrilateral are equal.
However, nothing is specified about the fourth side, which need not be equal (so, the angles between the sides may vary).
Thus, statement (2) ALONE is not sufficient.
When considered together, the two statements tell us that the 4 sides of the quadrilateral are equal making it a parallelogram (in fact, a rhombus which is a parallelogram with 4 equal sides).
Thus, BOTH statements TOGETHER are sufficient.

8. If five more scouts are added to a scout camp group, would the scout leader be able to divide the group into teams of 10 scouts each without any remainder?
(1) Initially, the number of scouts in the group was not divisible by ten.
(2) If 25 scouts are added to the group, the group can be divided into teams of 10 without any remainder.
• Statement (1) ALONE is sufficient, but statement (2) ALONE is not sufficient.
• Statement (2) ALONE is sufficient, but statement (1) ALONE is not sufficient.
• BOTH statements TOGETHER are sufficient, but NEITHER statement ALONE is sufficient.
• EACH statement ALONE is sufficient.
• Statements (1) and (2) TOGETHER are NOT sufficient.
Answer: Statement (2) ALONE is sufficient, but statement (1) ALONE is not sufficient.
Statement (1) does not guarantee that the new group can be divided into teams of 10 each.
For example, the group could have had 16 scouts initially, in which case the new group (of 21 scouts) cannot be divided into teams of 10 scouts each without any remainder.
On the other hand, the group could have had 15 scouts initially, in which case the new group (of 20 scouts) can be divided into teams of 10 scouts each without any remainder.
Thus, statement (1) ALONE is not sufficient.
Statement (2) implies that the number of scouts in the group initially is 5 less than a multiple of 10.
So, if 5 scouts are now added to the group, the new group can be divided into teams of 10 scouts each.
Thus, statement (2) ALONE is sufficient.

9. Is x + y even?
(1) xy is even.
(2) y / x is odd.
• Statement (1) ALONE is sufficient, but statement (2) ALONE is not sufficient.
• Statement (2) ALONE is sufficient, but statement (1) ALONE is not sufficient.
• BOTH statements TOGETHER are sufficient, but NEITHER statement ALONE is sufficient.
• EACH statement ALONE is sufficient.
• Statements (1) and (2) TOGETHER are NOT sufficient.
Answer: Statement (2) ALONE is sufficient, but statement (1) ALONE is not sufficient.
For (x + y) to be an even number, both x and y must be even or both must be odd.
Statement (1) specifies that xy is even; then either x or y must be even, but both need not necessarily be even.
For example, 2 x 3 = 6 (even number), then 2 + 3 = 5 (odd number).
However, 2 x 2 = 4 (even number), then 2 + 2 = 4 (even number).
∴ If the product is even, then the sum can be either odd or even.
Thus, statement (1) ALONE is not sufficient.
Statement (2) specifies that y / x is odd; then both x and y must be odd or both must be even.
For example, 15 / 3 = 5 (odd number), then 15 + 3 = 18 (even number).
Also, 14 / 2 = 7 (odd number), then 14 + 2 = 16 (even number).
∴ If the quotient is odd, then the sum is necessarily even as proven below.
Since (y / x) is odd, y / x = 2m + 1, where m is an integer.
Multiplying both sides by x, one obtains y = 2mx + x .
Adding x to both sides, x + y = 2mx + 2x.
Since the right-hand side is divisible by 2, it is clear that (x + y) is even.
Thus, statement (2) ALONE is sufficient.

10. If z is an integer, is 4 + z = 2?
(1) z < −2.
(2) z < 0.
• Statement (1) ALONE is sufficient, but statement (2) ALONE is not sufficient.
• Statement (2) ALONE is sufficient, but statement (1) ALONE is not sufficient.
• BOTH statements TOGETHER are sufficient, but NEITHER statement ALONE is sufficient.
• EACH statement ALONE is sufficient.
• Statements (1) and (2) TOGETHER are NOT sufficient.
Answer: Statement (1) ALONE is sufficient, but statement (2) ALONE is not sufficient.
Statement (1) specifies that z < −2.
Adding 4 to both sides of the inequality, one obtains 4 + z < 2.
Clearly, (4 + z) cannot be equal to 2.
Thus, statement (1) ALONE is sufficient.
Statement (2) specifies that z < 0.
Adding 4 to both sides of the inequality, we get 4 + z < 4.
However, the value of (4 + z) may or may not be 2.
Thus, statement (2) ALONE is not sufficient.
Alternatively, solving 4 + z = 2 gives z = −2. So, the question may be re-worded as 'Is z = −2?'
Statement (1) ALONE is sufficient, because it specifies z ≠ −2 (since z < −2).
Statement (2) ALONE is not sufficient, because z may be any negative integer (not necessarily −2).

11. What is the average of 7x and 91y?
(1) x + 13y = 100.
(2) x + y = 88.
• Statement (1) ALONE is sufficient, but statement (2) ALONE is not sufficient.
• Statement (2) ALONE is sufficient, but statement (1) ALONE is not sufficient.
• BOTH statements TOGETHER are sufficient, but NEITHER statement ALONE is sufficient.
• EACH statement ALONE is sufficient.
• Statements (1) and (2) TOGETHER are NOT sufficient.
Answer: Statement (1) ALONE is sufficient, but statement (2) ALONE is not sufficient.
Statement (1) specifies the value of (x + 13y).
So, the required average is (7x + 91y) / 2 = 7 (x + 13y) / 2 = 7 x 100 / 2 = 350.
Thus, statement (1) ALONE is sufficient.
Statement (2) specifies the value of (x + y), from which it is not possible to evaluate (7x + 91y).
Thus, statement (2) ALONE is not sufficient.

12. What is the value of the two-digit number?
(1) The product of its digits is 7.
(2) The larger of the two digits is in the units place.
• Statement (1) ALONE is sufficient, but statement (2) ALONE is not sufficient.
• Statement (2) ALONE is sufficient, but statement (1) ALONE is not sufficient.
• BOTH statements TOGETHER are sufficient, but NEITHER statement ALONE is sufficient.
• EACH statement ALONE is sufficient.
• Statements (1) and (2) TOGETHER are NOT sufficient.
Answer: BOTH statements TOGETHER are sufficient, but NEITHER statement ALONE is sufficient.
Statement (1) specifies that the product of the two digits is 7.
Since 7 is a prime number, the two digits are 1 and 7.
The required two-digit number could be either 17 or 71.
Thus, statement (1) ALONE is not sufficient.
Statement (2) specifies that the larger of the two digits is in the units place. There are many two-digit numbers that satisfy this criterion.
Thus, statement (2) ALONE is not sufficient.
When both statements are taken together, the required number is 17 (since the larger of the two digits is in the units place) and not 71.
Thus, BOTH statements TOGETHER are sufficient.

13. If p < 100 and is the cube of a positive integer, what is the value of p ?
(1) p is even.
(2) p is the square of an integer.
• Statement (1) ALONE is sufficient, but statement (2) ALONE is not sufficient.
• Statement (2) ALONE is sufficient, but statement (1) ALONE is not sufficient.
• BOTH statements TOGETHER are sufficient, but NEITHER statement ALONE is sufficient.
• EACH statement ALONE is sufficient.
• Statements (1) and (2) TOGETHER are NOT sufficient.
Answer: BOTH statements TOGETHER are sufficient, but NEITHER statement ALONE is sufficient.
The integers whose cubes are less than 100 are 1, 2, 3 and 4.
13 = 1; 23 = 8; 33 = 27; 43 = 64. So, p is 1, 8, 27 or 64.
Statement (1) indicates that p is either 8 or 64 (since 8 and 64 are even, whereas 1 and 27 are odd).
Thus, statement (1) ALONE is not sufficient.
Statement (2) indicates that p is either 1 or 64 (since 12 = 1 and 82 = 64).
Thus, statement (2) ALONE is not sufficient.
However, when both statements are considered together, p = 64 (since it is less than 100, the cube of 4, the square of 8, and even).
Thus, BOTH statements TOGETHER are sufficient.

14. Janice spent $200 at a shopping mall. How many items worth $40 did she purchase?
(1) All the items she purchased were worth either $40 or $80.
(2) She purchased more than one item worth $80.
• Statement (1) ALONE is sufficient, but statement (2) ALONE is not sufficient.
• Statement (2) ALONE is sufficient, but statement (1) ALONE is not sufficient.
• BOTH statements TOGETHER are sufficient, but NEITHER statement ALONE is sufficient.
• EACH statement ALONE is sufficient.
• Statements (1) and (2) TOGETHER are NOT sufficient.
Answer: BOTH statements TOGETHER are sufficient, but NEITHER statement ALONE is sufficient.
Statement (1) indicates that the money could have been spent in the following three ways:
(a) Five items worth $40.
(b) One item worth $80 and three items worth $40.
(c) Two items worth $80 and one item worth $40.
Thus, statement (1) ALONE is not sufficient.
Statement (2) simply states that more than one item worth $80 was purchased.
Thus, statement (2) ALONE is not sufficient.
When both statements are combined, it may be concluded that two items worth $80 and one item worth $40 were purchased.Thus, BOTH
statements TOGETHER are sufficient.

15. A line passes through the origin in the coordinate plane and the point (a, b). What is the slope of the line?
(1) a = 3b
(2) a = 6.3
• Statement (1) ALONE is sufficient, but statement (2) ALONE is not sufficient.
• Statement (2) ALONE is sufficient, but statement (1) ALONE is not sufficient.
• BOTH statements TOGETHER are sufficient, but NEITHER statement ALONE is sufficient.
• EACH statement ALONE is sufficient.
• Statements (1) and (2) TOGETHER are NOT sufficient.
Answer: Statement (1) ALONE is sufficient, but statement (2) ALONE is not sufficient.
Since the line passes through the origin (0, 0) and the point (a, b), its slope is b/a.
Statement (1) specifies a = 3b, which gives the the slope b/a = 1/3.
Thus, statement (1) ALONE is sufficient.
Statement (2) specifies only a. Without knowing b, the slope cannot be determined.
Thus, statement (2) ALONE is not sufficient.

  Try the Quiz :     GMAT Test Prep : Quantitative Math Data Sufficiency Test I


Contact Info © 1999-2024 Syvum Technologies Inc. Privacy Policy Disclaimer and Copyright
Previous
-
Next
-